¿Qué carga utilizar en la condición de cuantificación de Dirac?

Tengo una pregunta de seguimiento a los monopolos magnéticos de Dirac y la cuantificación de la carga eléctrica fraccionaria de quarks , con respecto a si la "unidad de carga eléctrica" ​​en la condición de cuantificación de Dirac debe ser mi o mi 3 debido a la carga fraccionaria de los quarks. La respuesta de Ben Crowell argumenta que deberías usar mi , porque los quarks están confinados, por lo que nunca podría tomar un solo quark en un bucle de Wilson alrededor de la cuerda de Dirac. Pero por encima de la temperatura de Hagedorn , los quarks se desconfinan, así que ¿no podrías realizar este experimento a una temperatura superior a la temperatura de Hagedorn? ¿Significa esto que deberíamos triplicar la carga mínima permitida del monopolo magnético?

Respuestas (1)

La respuesta se puede encontrar en la sección 6 de http://www.sciencedirect.com/science/article/pii/0550321381902376 . Por encima de la temperatura de Hagedorn, la interacción fuerte se vuelve de largo alcance y cae como 1 / r , al igual que la interacción de Coulomb. Mientras trenzas una carga- mi / 3 quark alrededor de una cuerda de Dirac, toma una fase de Aharonov-Bohm de la carga eléctrica del quark que rodea el flujo magnético de la cuerda, pero también toma una fase de Aharonov-Bohm adicional de los tubos de color de la interacción fuerte que cruzan la cuerda de Dirac, y convierte descubra que la fase Aharonov-Bohm de la interacción fuerte es exactamente el doble que la fase electromagnética Aharonov-Bohm, lo que hace que la fase total sea equivalente a lo que habría obtenido si hubiera trenzado una partícula de carga de color neutro mi (como un electrón) en lugar de un quark. La fuerte interacción esencialmente compensa la "falta" ( 2 / 3 ) mi de carga eléctrica. Por lo tanto, la unidad correcta de carga eléctrica para usar en la condición de cuantificación de Dirac es mi , no mi / 3 .

De hecho, incluso puede mover un solo quark alrededor del monopolo en la fase confinada; solo necesita moverse en un círculo más pequeño que el radio de confinamiento. Entonces también ocurre exactamente el mismo efecto que el descrito anteriormente.

Ver también pág. 468 de http://www.theory.caltech.edu/~preskill/pubs/preskill-1984-monopoles.pdf .